Practice Number series - quantitative aptitude Online Quiz (set-1) For All Competitive Exams

Q-1)   What should come in place of the question mark (?) in the following number series?

8, 15, 28, 53, ?

(a)

(b)

(c)

(d)

(e)

Explanation:

Let x = 8

then 15 = 2x – 1 = y

28 = 2y – 2 = z

53 = 2z – 3 = m

Next term in the pattern should be

2m – 4 = 2 × 53 – 4

= 102


Q-2)   What should come in place of the question mark (?) in the following number serie?

250, 100, 40, ?, 6.4, 2.56

(a)

(b)

(c)

(d)

(e)

Explanation:

The given number series is based on the following pattern:

$250_{÷2.5}$ $100_{÷2.5}$ $40_{÷2.5}$ $16_{÷2.5}$ $6.4_{÷2.5}$ 2.56

Hence, the number 16 will replace the question mark.


Q-3)   What should come in place of the question mark (?) in the following number series?

4, 15, 16, ?, 36, 63, 64

(a)

(b)

(c)

(d)

(e)

Explanation:

Pattern is $2^2$, $4^2$ – 1,$4^2$, $6^2$ – 1, $6^2$ and so on.


Q-4)   A wrong number is given in the following number series. Find out that wrong number.

2, 11, 38, 197, 1172, 8227, 65806

(a)

(b)

(c)

(d)

(e)

Explanation:

The series is based on the following pattern:

2 × 3 + 5 = 11

11 × 4 – 6 = 38

38 × 5 + 7 = 197

197 × 6 – 8 = 1174 ; not 1172

1174 × 7 + 9 = 8227

8227 × 8 – 10 = 65806

Clearly, 1172 is the wrong number and it should be replaced by 1174.


Q-5)   What should come in place of the question mark (?) in the following number series?

2, 3, 10, 15, 26, ?, 55

(a)

(b)

(c)

(d)

(e)

Explanation:

The series exhibits the pattern of $n^2$ + 1, $n^2$ – 1,

alternatively, n taking values 1, 2.............1


Q-6)   What should come in place of the question mark (?) in the following number serie?

3, 7, 13, ?, 31, 43

(a)

(b)

(c)

(d)

(e)

Explanation:

The given number series is based on the following pattern:

7 + 6 = 13

13 + 8 = 21

21 + 10 = 31

31 + 12 = 43

Hence, the number 21 will replace the question mark.


Q-7)   What should come in place of the question mark (?) in the following number series?

64, 54, 69, 49, 74, 44, ?

(a)

(b)

(c)

(d)

(e)

Explanation:

The series is as follows:

64 + 5 = 69;

69 + 5 = 74;

74 + 5 = 79

54 – 5 = 49;

49 – 5 = 44


Q-8)   A wrong number is given in the following number series. Find out that wrong number.

16, 19, 21, 30, 46, 71, 107

(a)

(b)

(c)

(d)

(e)

Explanation:

The series is based on the following pattern :

16 + $1^2$ = 17; not 19

17 + $2^2$ = 21

21 + $3^2$ = 30

30 + $4^2$ = 46

46 + $5^2$ = 71

71 + $6^2$ = 107

Clearly, 19 should replaced by 17.


Q-9)   What should come in place of the question mark (?) in the following number series?

1, 8, 9, ?, 25, 216, 49

(a)

(b)

(c)

(d)

(e)

Explanation:

Can you see that the pattern is

$1^2$, $2^3$, $3^2$, $4^3$, $5^2$, $6^3$, $7^2$


Q-10)   A wrong number is given in the following number series. Find out that wrong number.

7, 9, 16, 25, 41, 68, 107, 173

(a)

(b)

(c)

(d)

(e)

Explanation:

The series is based on the following pattern :

7 + 9 = 16

9 + 16 = 25

16 + 25 = 41

25 + 41 = 66; 68

41 + 66 = 107

66 + 107 = 173

Clearly, 68 should be replaced by 66


Q-11)   What should come in place of the question mark (?) in the following number serie?

4, 16, 36, 64, 100, ?

(a)

(b)

(c)

(d)

(e)

Explanation:

The series is as follows

$(2)^2$, $(4)^2$, $(6)^2$, $(8)^2$, $(10)^2$, $(12)^2$

Hence, ? = $(12)^2$ = 144


Q-12)   Only one number is wrong in the following number series. Find out the wrong number.

48, 72, 108, 162, 243, 366

(a)

(b)

(c)

(d)

(e)

Explanation:

The series is × 1.5

The wrong number is 366

It should be 243 × 1.5 = 364.5


Q-13)   What should come in place of the question mark (?) in the following number series?

5, 5, 15, 75, ?, 4725, 51975

(a)

(b)

(c)

(d)

(e)

Explanation:

The series is as follows:

× 1, × 3, × 5, × 7, × 9, × 11

Hence, ? = 75 × 7 = 525


Q-14)   Only one number is wrong in the following number series. Find out the wrong number.

8, 27, 64, 125, 218, 343

(a)

(b)

(c)

(d)

(e)

Explanation:

The series is $(2)^3$, $(3)^3$, $(4)^3$, $(5)^3$, $(6)^3$, $(7)^3$,

The wrong number is 218

It should be $(6)^3$ = 216


Q-15)   What should come in place of the question mark (?) in the following number serie?

9, 27, 36, 63, 99, ?

(a)

(b)

(c)

(d)

(e)

Explanation:

Here,

9 + 27 = 36

27 + 36 = 63

36 + 63 = 99

63 + 99 = 162.


Q-16)   What should come in place of the question mark (?) in the following number series?

697, 553, 453, 389, 353, (?)

(a)

(b)

(c)

(d)

(e)

Explanation:

The series is as follows:

– $12^2$, – $10^2$, – $8^2$, – $6^2$, – $4^2$

Hence, ? = 353 – $4^2$ = 337


Q-17)   What should come in place of the question mark (?) in the following number series?

588, 587, 583, 574, 558, ?, 497

(a)

(b)

(c)

(d)

(e)

Explanation:

The series is as follows:

– $1^2$, – $2^2$, –$3^2$, – $4^2$, – $5^2$, – $6^2$

Hence, ? = 558 – $5^2$ = 533


Q-18)   What should come in place of the question mark (?) in the following number series?

52, 26, 26, 39, 78, ?, 585

(a)

(b)

(c)

(d)

(e)

Explanation:

The series is as follows:

× $1/2$, × 1, × 1$1/2$, × 2, × 2$1/2$, × 3

Hence, ? = 78 × 2$1/2$ = 195


Q-19)   What should come in place of the question mark (?) in the following number series?

119, 131, 155, 191, 239, (?)

(a)

(b)

(c)

(d)

(e)

Explanation:

The series is as follows:

+ (12 × 1), + (12 × 2), + (12 × 3), + (12 × 4), + (12 × 5)

Hence, ? = 239 + (12 × 5) = 299


Q-20)   What should come in place of the question mark (?) in the following number series?

336, 224, 168, 140, 126, (?)

(a)

(b)

(c)

(d)

(e)

Explanation:

The series is as follows:

– 112, – 56, – 28, – 14, – 7

Hence, ? = 126 – 7 = 119